LSAT and Law School Admissions Forum

Get expert LSAT preparation and law school admissions advice from PowerScore Test Preparation.

User avatar
 Dave Killoran
PowerScore Staff
  • PowerScore Staff
  • Posts: 5852
  • Joined: Mar 25, 2011
|
#85106
Complete Question Explanation
(The complete setup for this game can be found here: lsat/viewtopic.php?f=351&t=3899)

The correct answer choice is (A)

Note that this is a Justify question, wherein you must supply the answer choice that forces the desired result, which in this case is to have only one possible solution.

Looking over our setup, when we added the contrapositive of the third rule to the original first rule, the following resulted:

G2-Q9-d1.png

Thus, if an answer stipulated that light 1 was purple, that would result in a single color sequence. As it happens, answer choice (A) indicates that light 1 is purple, and answer choice (A) is thus correct.

Note that this question can also be easily solved using hypotheticals.
You do not have the required permissions to view the files attached to this post.
 wwarui
  • Posts: 32
  • Joined: Nov 13, 2011
|
#13250
Hello,

Question Number 9

Please explain this question and illustrate why answer A is correct. I have tried but it does not seem to work. Why would the colour sequence be purple when it clearly states in the stimulus that "Whenever light 1 is purple, light 2 is yellow".
Thanks!!
 Emily Haney-Caron
PowerScore Staff
  • PowerScore Staff
  • Posts: 577
  • Joined: Jan 12, 2012
|
#13266
Hello,

Here, we're looking for an answer where we have only one possible solution to the game. The question isn't asking for an answer where all three lights will be the same color; rather, we're just trying to find an answer that only leaves one possibility for the other two light colors.

A does that. If light 1 is purple, we know from Rule 1 that light 2 is yellow. We also know from rule 3 that if light 1 is purple, then light 3 cannot be purple or yellow, since rule 3 would then require that light 2 be purple (but we know light 2 is yellow). Therefore, light 3 has to be green if light 1 is purple.

Therefore, if light 1 is purple, the sequence is P-Y-G.
 wwarui
  • Posts: 32
  • Joined: Nov 13, 2011
|
#13267
Hi Emily:

Gosh! I completely misunderstood the question. Thank you for clarifying this. No wonder I couldn't figure it out.
THANKS!!!

Get the most out of your LSAT Prep Plus subscription.

Analyze and track your performance with our Testing and Analytics Package.